Infinite Arccotangents Equals One Arctangent

Geometry Level 4

arccot ( 1 2 + 3 4 ) + arccot ( 2 2 + 3 4 ) + arccot ( 3 2 + 3 4 ) + arccot ( 4 2 + 3 4 ) + = ? \text{arccot} \left(1^2 + \dfrac34\right) + \text{arccot} \left(2^2 + \dfrac34\right) + \text{arccot} \left(3^2 + \dfrac34\right) + \text{arccot} \left(4^2 + \dfrac34\right) + \cdots = \, ?

arctan 1 \text{arctan} 1 arctan 2 \text{arctan} 2 arctan 3 \text{arctan} 3 arctan 4 \text{arctan} 4

This section requires Javascript.
You are seeing this because something didn't load right. We suggest you, (a) try refreshing the page, (b) enabling javascript if it is disabled on your browser and, finally, (c) loading the non-javascript version of this page . We're sorry about the hassle.

2 solutions

Rishabh Jain
Jul 10, 2016

Relevant wiki: Sum and Difference Trigonometric Formulas - Problem Solving

T = r = 1 n cot 1 ( r 2 + 3 4 ) = r = 1 n tan 1 ( 1 r 2 + 3 4 ) = r = 1 n tan 1 ( 1 1 + ( r 2 1 4 ) ) = r = 1 n tan 1 ( ( r + 1 2 ) ( r 1 2 ) 1 + ( r + 1 2 ) ( r 1 2 ) ) = r = 1 n [ tan 1 ( r + 1 2 ) tan 1 ( r 1 2 ) ] \begin{aligned}\mathfrak{T}=&\displaystyle\sum_{r=1}^n\cot^{-1}\left(r^2+\dfrac 34\right)\\=&\displaystyle\sum_{r=1}^n\tan^{-1}\left( \dfrac{1}{r^2+\dfrac 34}\right)\\=&\displaystyle\sum_{r=1}^n\tan^{-1} \left(\dfrac{1}{ 1+\left(r^2-\dfrac 14\right)}\right)\\=&\displaystyle\sum_{r=1}^n\tan^{-1}\left( \dfrac{\left(r+\frac 12\right)-\left(r-\frac 12\right)}{1+\left(r+\frac 12\right)\left(r-\frac 12\right)}\right)\\=&\displaystyle\sum_{r=1}^n\left[\tan^{-1}\left(r+\frac 12\right)-\tan^{-1}\left(r-\frac 12\right)\right]\end{aligned}

( A T e l e s c o p i c S e r i e s ) \large\mathbf{(A~Telescopic~Series)}

= tan 1 ( n + 1 2 ) tan 1 1 2 \large=\tan^{-1}\left(n+\frac 12\right)-\tan^{-1}\frac 12

lim n T = π 2 tan 1 1 2 = tan 1 2 \color{#0C6AC7}{\large\therefore\displaystyle\lim_{n\to\infty}\mathfrak{T}=\dfrac{\pi}{2}-\tan^{-1}\frac 12=\tan^{-1}2}

Hung Woei Neoh
Jul 10, 2016

cot 1 ( 1 2 + 3 4 ) + cot 1 ( 2 2 + 3 4 ) + cot 1 ( 3 2 + 3 4 ) + cot 1 ( 4 2 + 3 4 ) + = lim n k = 1 n cot 1 ( 4 k 2 + 3 4 ) = lim n k = 1 n tan 1 ( 4 4 k 2 + 3 ) = lim n tan 1 ( 4 n 5 + 2 n ) = lim n tan 1 ( 4 5 n + 2 ) = tan 1 ( 4 0 + 2 ) = tan 1 2 \cot^{-1}\left(1^2+\dfrac{3}{4}\right)+\cot^{-1}\left(2^2+\dfrac{3}{4}\right)+\cot^{-1}\left(3^2+\dfrac{3}{4}\right)+\cot^{-1}\left(4^2+\dfrac{3}{4}\right)+\ldots\\ =\displaystyle \lim_{n \to \infty} \sum_{k=1}^n \cot^{-1}\left(\dfrac{4k^2+3}{4}\right)\\ =\displaystyle \lim_{n \to \infty} \sum_{k=1}^n \color{#3D99F6}{\tan^{-1}\left(\dfrac{4}{4k^2+3}\right)}\\ =\displaystyle\lim_{n \to \infty} \color{#D61F06}{\tan^{-1}\left(\dfrac{4n}{5+2n}\right)}\\ =\displaystyle\lim_{n \to \infty} \tan^{-1}\left(\dfrac{4}{\frac{5}{n}+2}\right)\\ =\tan^{-1} \left(\dfrac{4}{0+2}\right)\\ =\boxed{\tan^{-1} 2}

The proof for both highlighted expressions are found below


Proof 1:

cot 1 a = tan 1 ( 1 a ) \color{#3D99F6}{\cot^{-1} a = \tan^{-1}\left(\dfrac{1}{a}\right)}

Let cot 1 a = b \cot^{-1} a= b

a = cot b a = 1 tan b tan b = 1 a b = tan 1 ( 1 a ) cot 1 a = tan 1 ( 1 a ) a=\cot b\\ a=\dfrac{1}{\tan b}\\ \tan b= \dfrac{1}{a}\\ b = \tan^{-1}\left(\dfrac{1}{a}\right)\\ \color{#3D99F6}{\cot^{-1} a =\tan^{-1}\left(\dfrac{1}{a}\right)}


Proof 2:

k = 1 n tan 1 ( 4 4 k 2 + 3 ) = tan 1 ( 4 n 5 + 2 n ) \color{#D61F06}{\sum_{k=1}^n \tan^{-1}\left(\dfrac{4}{4k^2+3}\right)=\tan^{-1}\left(\dfrac{4n}{5+2n}\right)}

First of all, we need to show that:

tan 1 a + tan 1 b = tan 1 ( a + b 1 a b ) \color{#20A900}{\tan^{-1} a+ \tan^{-1} b = \tan^{-1}\left(\dfrac{a+b}{1-ab}\right)}

Let tan 1 a = x , tan 1 b = y \tan^{-1} a = x,\;\tan^{-1} b = y

tan 1 a + tan 1 b = x + y tan ( x + y ) = tan ( tan 1 a + tan 1 b ) tan ( x + y ) = tan ( tan 1 a ) + tan ( tan 1 b ) 1 tan ( tan 1 a ) tan ( tan 1 b ) tan ( x + y ) = a + b 1 a b x + y = tan 1 ( a + b 1 a b ) tan 1 a + tan 1 b = tan 1 ( a + b 1 a b ) \tan^{-1} a+ \tan^{-1} b = x+y\\ \tan(x+y) = \tan\left(\tan^{-1} a+ \tan^{-1} b\right)\\ \tan(x+y)=\dfrac{\tan\left(\tan^{-1} a\right)+\tan\left( \tan^{-1} b\right)}{1-\tan\left(\tan^{-1} a\right)\tan\left( \tan^{-1} b\right)}\\ \tan(x+y)=\dfrac{a+b}{1-ab}\\ x+y=\tan^{-1}\left(\dfrac{a+b}{1-ab}\right)\\ \color{#20A900}{\tan^{-1} a+ \tan^{-1} b = \tan^{-1}\left(\dfrac{a+b}{1-ab}\right)}

To see this pattern, let us try to get the sum for n = 1 , 2 , 3 , 4 n=1,2,3,4

n = 1 k = 1 1 tan 1 ( 4 4 k 2 + 3 ) = tan 1 ( 4 7 ) n = 2 k = 1 2 tan 1 ( 4 4 k 2 + 3 ) = tan 1 ( 4 7 ) + tan 1 ( 4 19 ) = tan 1 ( 4 7 + 4 19 1 4 7 × 4 19 ) = tan 1 ( 8 9 ) n = 3 k = 1 3 tan 1 ( 4 4 k 2 + 3 ) = tan 1 ( 4 7 ) + tan 1 ( 4 19 ) + tan 1 ( 4 39 ) = tan 1 ( 8 9 ) + tan 1 ( 4 39 ) = tan 1 ( 8 9 + 4 39 1 8 9 × 4 39 ) = tan 1 ( 12 11 ) n = 4 k = 1 4 tan 1 ( 4 4 k 2 + 3 ) = tan 1 ( 4 7 ) + tan 1 ( 4 19 ) + tan 1 ( 4 39 ) + tan 1 ( 4 67 ) = tan 1 ( 12 11 ) + tan 1 ( 4 67 ) = tan 1 ( 12 11 + 4 67 1 12 11 × 4 67 ) = tan 1 ( 16 13 ) n=1 \implies\\ \displaystyle\sum_{k=1}^1 \tan^{-1}\left(\dfrac{4}{4k^2+3}\right)\\ =\tan^{-1}\left(\dfrac{4}{7}\right)\\ n=2 \implies\\ \displaystyle\sum_{k=1}^2 \tan^{-1}\left(\dfrac{4}{4k^2+3}\right)\\ =\tan^{-1}\left(\dfrac{4}{7}\right)+\tan^{-1}\left(\dfrac{4}{19}\right)\\ =\tan^{-1}\left(\dfrac{\frac{4}{7}+\frac{4}{19}}{1-\frac{4}{7}\times\frac{4}{19}}\right)\\ =\tan^{-1}\left(\dfrac{8}{9}\right)\\ n=3 \implies\\ \displaystyle\sum_{k=1}^3 \tan^{-1}\left(\dfrac{4}{4k^2+3}\right)\\ =\color{#69047E}{\tan^{-1}\left(\dfrac{4}{7}\right)+\tan^{-1}\left(\dfrac{4}{19}\right)}+\tan^{-1}\left(\dfrac{4}{39}\right)\\ =\tan^{-1}\left(\dfrac{8}{9}\right)+\tan^{-1}\left(\dfrac{4}{39}\right)\\ =\tan^{-1}\left(\dfrac{\frac{8}{9}+\frac{4}{39}}{1-\frac{8}{9}\times\frac{4}{39}}\right)\\ =\tan^{-1}\left(\dfrac{12}{11}\right)\\ n=4 \implies\\ \displaystyle\sum_{k=1}^4 \tan^{-1}\left(\dfrac{4}{4k^2+3}\right)\\ =\color{#69047E}{\tan^{-1}\left(\dfrac{4}{7}\right)+\tan^{-1}\left(\dfrac{4}{19}\right)+\tan^{-1}\left(\dfrac{4}{39}\right)}+\tan^{-1}\left(\dfrac{4}{67}\right)\\ =\tan^{-1}\left(\dfrac{12}{11}\right)+\tan^{-1}\left(\dfrac{4}{67}\right)\\ =\tan^{-1}\left(\dfrac{\frac{12}{11}+\frac{4}{67}}{1-\frac{12}{11}\times\frac{4}{67}}\right)\\ =\tan^{-1}\left(\dfrac{16}{13}\right)

Notice that for the first 4 4 values of n n , the sum gives you tan 1 ( 4 n 5 + 2 n ) \tan^{-1}\left(\dfrac{4n}{5+2n}\right)

This relationship can be further proved with induction.

Proof by induction

Let P n P_n be the proposition that k = 1 n tan 1 ( 4 4 k 2 + 3 ) = tan 1 ( 4 n 5 + 2 n ) \displaystyle\color{#D61F06}{\sum_{k=1}^n \tan^{-1}\left(\dfrac{4}{4k^2+3}\right)=\tan^{-1}\left(\dfrac{4n}{5+2n}\right)} for all positive integers n n .

Base case: Let n = 1 n=1

LHS = tan 1 ( 4 4 ( 1 ) 2 + 3 ) = tan 1 ( 4 7 ) =\tan^{-1}\left(\dfrac{4}{4(1)^2+3}\right)=\tan^{-1}\left(\dfrac{4}{7}\right)

RHS = tan 1 ( 4 ( 1 ) 5 + 2 ( 1 ) ) = tan 1 ( 4 7 ) =\tan^{-1}\left(\dfrac{4(1)}{5+2(1)}\right)=\tan^{-1}\left(\dfrac{4}{7}\right)

LHS = RHS. Base case holds true.

Induction Hypothesis: Suppose P m P_m is true for some integer m m . Consider P m + 1 P_{m+1}

LHS P m + 1 P_{m+1}

= tan 1 ( 4 m 5 + 2 m ) + tan 1 ( 4 4 ( m + 1 ) 2 + 3 ) = tan 1 ( 4 m 5 + 2 m + 4 4 m 2 + 8 m + 7 1 ( 4 m 5 + 2 m ) ( 4 4 m 2 + 8 m + 7 ) ) = tan 1 ( ( 4 m ( 4 m 2 + 8 m + 7 ) + 4 ( 5 + 2 m ) ( 5 + 2 m ) ( 4 m 2 + 8 m + 7 ) ) ( ( 5 + 2 m ) ( 4 m 2 + 8 m + 7 ) 16 m ( 5 + 2 m ) ( 4 m 2 + 8 m + 7 ) ) ) = tan 1 ( 16 m 3 + 32 m 2 + 36 m + 20 8 m 3 + 36 m 2 + 38 m + 35 ) = tan 1 ( 4 ( 4 m 3 + 8 m 2 + 9 m + 5 ) 8 m 3 + 28 m 2 + 8 m 2 + 28 m + 10 m + 35 ) = tan 1 ( 4 ( 4 m 3 + 4 m 2 + 4 m 2 + 4 m + 5 m + 5 ) 4 m 2 ( 2 m + 7 ) + 4 m ( 2 m + 7 ) + 5 ( 2 m + 7 ) ) = tan 1 ( 4 ( 4 m 2 ( m + 1 ) + 4 m ( m + 1 ) + 5 ( m + 1 ) ) ( 2 m + 7 ) ( 4 m 2 + 4 m + 5 ) ) = tan 1 ( 4 ( m + 1 ) ( 4 m 2 + 4 m + 5 ) ( 5 + 2 m + 2 ) ( 4 m 2 + 4 m + 5 ) ) = tan 1 ( 4 ( m + 1 ) 5 + 2 ( m + 1 ) ) =\tan^{-1}\left(\dfrac{4m}{5+2m}\right)+\tan^{-1}\left(\dfrac{4}{4(m+1)^2+3}\right)\\ =\tan^{-1}\left(\dfrac{\frac{4m}{5+2m}+\frac{4}{4m^2+8m+7}}{1-\left(\frac{4m}{5+2m}\right)\left(\frac{4}{4m^2+8m+7}\right)}\right)\\ =\tan^{-1}\left(\dfrac{\left(\frac{4m(4m^2+8m+7)+4(5+2m)}{(5+2m)(4m^2+8m+7)}\right)}{\left(\frac{(5+2m)(4m^2+8m+7)-16m}{(5+2m)(4m^2+8m+7)}\right)}\right)\\ =\tan^{-1}\left(\dfrac{16m^3+32m^2+36m+20}{8m^3+36m^2+38m+35}\right)\\ =\tan^{-1}\left(\dfrac{4(4m^3+8m^2+9m+5)}{8m^3+28m^2+8m^2+28m+10m+35}\right)\\ =\tan^{-1}\left(\dfrac{4(4m^3+4m^2+4m^2+4m+5m+5)}{4m^2(2m+7)+4m(2m+7)+5(2m+7)}\right)\\ =\tan^{-1}\left(\dfrac{4\left(4m^2(m+1)+4m(m+1)+5(m+1)\right)}{(2m+7)(4m^2+4m+5)}\right)\\ =\tan^{-1}\left(\dfrac{4(m+1)(4m^2+4m+5)}{(5+2m+2)(4m^2+4m+5)}\right)\\ =\tan^{-1}\left(\dfrac{4(m+1)}{5+2(m+1)}\right)

= = RHS P m + 1 P_{m+1}

Conclusion: By Mathematical Induction, since P 1 P_1 is true and P m P_m is true implies P m + 1 P_{m+1} is true, thus P n P_n is true for all positive integers n n

Phew! That was a long proof

Very nice solution.(+1) How do u align the text at center?

M D - 4 years, 10 months ago

0 pending reports

×

Problem Loading...

Note Loading...

Set Loading...